How does the magnetic field becomes 0 outside a toroid? Also how can we explain it with magnetic field lines?
Asked
Active
Viewed 317 times
2
-
1Draw a toroid with a tightly wound coil around it. Now apply Ampere's law. Why would you think that there is non-negligible field outside the toroid? – hyportnex Jul 12 '20 at 21:37
-
@hyportnex What are you saying, is there field present outside a toroid? – Harrison Wells Jul 12 '20 at 22:02
-
1You have to consider it as an ideal toroid. – Ken Jul 12 '20 at 22:23
-
Related : (1) Does magnetic field depend on z inside a toroidal coil?,(2) Magnetic Induction at the centre of a Toroid. – Frobenius Jul 12 '20 at 22:27
-
I ask, have you seen the reasoning for why the magnetic field is considered negligible outside a solenoid? If so you can apply the same reasoning to a toroidal coil. – Triatticus Jul 13 '20 at 00:47
-
@Triatticus For that the radius of the toroid must be very less than its length. – Harrison Wells Jul 13 '20 at 06:04
-
Related: https://physics.stackexchange.com/questions/328655/does-a-dc-supplied-superconductive-coil-gives-off-radiation – elias2010 Jul 13 '20 at 08:10
-
@HarrisonWells what I mean is in essence exactly what hyportnex mentioned, it's the same proof and doesn't matter the specific dimensions of the torus as you're just interested in a small segment of it anyways. – Triatticus Jul 13 '20 at 16:08
1 Answers
0
You might have heard of amperes circuit law which states that integral of B.dl is equal to current enclosed in the loop and we see that outside a toroid we can't find any current enclosed so magnetic field is 0
-
Nope, you can't say that for sure! The looped line integral is 0 (and not the magnetic field). Hence, it can't be said that magnetic field is 0 as the integral is looped. – Harrison Wells Jul 13 '20 at 13:58
-
You may want to describe where you are putting your chosen loop and the symmetry involved. – R.W. Bird Jul 13 '20 at 15:07